A set consists of of several different integers. Is the pr..

This topic has expert replies
Junior | Next Rank: 30 Posts
Posts: 25
Joined: Sun Nov 23, 2008 11:21 am
A set consists of of several different integers. Is the product of all the integers in the list positive?

(1) The product of the greatest and smallest of the integers in the list is positive

(2) There are an even number of integers in the list

Junior | Next Rank: 30 Posts
Posts: 28
Joined: Sun Nov 02, 2008 12:35 pm
Thanked: 7 times

by sachinkr » Sun Jan 18, 2009 10:47 pm
Statement 1: Smallest & greatest number product is Positive. So both these numbers are either Positive or Negative. (-10,-5,...,-1) or (3,6,... 10) But as we don't know the number of elements in the set, we cannot be sure if the product of all numbers will be Positive or Negative - Insufficient

Statement 2: A set contains even number of elements, can have product of all numbers either +/-.

Combining both the statements:
The set of integers are either all positive and even or Negative and even. The product will always be positive. Sufficient

Ans (C)

Junior | Next Rank: 30 Posts
Posts: 25
Joined: Sun Nov 23, 2008 11:21 am

by Emawk » Mon Jan 19, 2009 12:11 am
Let's say the set has 4 numbers:

Combining statements (1) and (2):

scenerio1:

2, -3, 4, 5 -- the product of these four numbers is a negative

scenario 2:

-2, 3, 4, -5 -- the product of these four numbers is positive

Both scenarios satisfy statement (1) and statement (2), but the answer could still go both ways.

I'm probably misinterpreting the question.
Last edited by Emawk on Mon Jan 19, 2009 10:32 am, edited 4 times in total.

User avatar
Legendary Member
Posts: 2134
Joined: Mon Oct 20, 2008 11:26 pm
Thanked: 237 times
Followed by:25 members
GMAT Score:730
(1) The product of the greatest and smallest of the integers in the list is positive

-1 -2 -3

1 2 3

Insuf

(2) There are an even number of integers in the list

-1 3

2 3

INSUF

1+2)

first statement tells us the numbers are either POSITIVE or NEGATIVE

Second statement tell us the ones in the middle can be PAIRS

1 2 3 4 = +

-1 -2 -3 - 4 = +

Choose C
LGTCH
---------------------
"DON'T LET ANYONE STEAL YOUR DREAM!"

Junior | Next Rank: 30 Posts
Posts: 25
Joined: Sun Nov 23, 2008 11:21 am

by Emawk » Mon Jan 19, 2009 12:26 am
logitech,

How did you derive from statement (2) that the integers in the middle can be pairs?

User avatar
Legendary Member
Posts: 2134
Joined: Mon Oct 20, 2008 11:26 pm
Thanked: 237 times
Followed by:25 members
GMAT Score:730

by logitech » Mon Jan 19, 2009 12:29 am
Emawk wrote:logitech,

How did you derive from statement (2) that the integers in the middle can be pairs?
(2) There are an even number of integers in the list

Since there are even numbers in the set

EVEN - (the first + the last) = ANOTHER EVEN number

So Even Number can be paired. In other words they are divisible by 2
LGTCH
---------------------
"DON'T LET ANYONE STEAL YOUR DREAM!"

Junior | Next Rank: 30 Posts
Posts: 25
Joined: Sun Nov 23, 2008 11:21 am

by Emawk » Mon Jan 19, 2009 1:13 am
logitech wrote:
Emawk wrote:logitech,

How did you derive from statement (2) that the integers in the middle can be pairs?
(2) There are an even number of integers in the list

Since there are even numbers in the set

EVEN - (the first + the last) = ANOTHER EVEN number

So Even Number can be paired. In other words they are divisible by 2
So statement (2) isn't saying n = even?

Senior | Next Rank: 100 Posts
Posts: 35
Joined: Thu Jun 05, 2008 8:53 am
Location: Israel
Thanked: 2 times

by yvichman » Mon Jan 19, 2009 9:15 am
Im confused also.

Statement 1 tells us that the product of the highest and lowest is Positive
so lets say our numbers are :

1 2 3 4 1 and 4 have the same sign, either both - or both + to give us a postive product

Statement 2 says there are an even # of numbers in the set.
So 1 2 3 4 still works here.
Why cant it be -2 +3 ? or +3 , -2? There are still an even # of numbers here

Junior | Next Rank: 30 Posts
Posts: 25
Joined: Sun Nov 23, 2008 11:21 am

by Emawk » Mon Jan 19, 2009 4:13 pm
logitec, would you be able to clarify statement (2) more?

Junior | Next Rank: 30 Posts
Posts: 28
Joined: Sun Nov 02, 2008 12:35 pm
Thanked: 7 times

by sachinkr » Tue Jan 20, 2009 1:12 pm
Emawk the conclusion from statement A is that All the numbers in the seris are either positive or Negative. ( From Product of Smallest and largest number is a positive number)

statement B : number of elements in set is even.

If we combine both the statements:
1. Seris contains all Positive or Negative ( so a set cannot contain have (-3,2 or 4,-6 etc)
Eg: -7,-4,-3,-2 ( All negative nos)
1,2,4,5,7,8 ( All positive nos)
2. Even number of integers

As all the numbers are either:
A. Positive and the set has even number of integers - Product will be positive
B. Negative nos and the set has even nos of integers - Product will be positive.

Hence (C) as the product will always be positive

User avatar
Master | Next Rank: 500 Posts
Posts: 176
Joined: Sun Feb 21, 2010 6:00 am
Thanked: 2 times
GMAT Score:710

by Pdgmat2010 » Sat May 29, 2010 5:11 am
Can someone shed some more light on this problem?

Are we supposed to assume that the list is in ascending order?

How else is C possible?

Please help!!

Cheers,
Pd

User avatar
Legendary Member
Posts: 1460
Joined: Tue Dec 29, 2009 1:28 am
Thanked: 135 times
Followed by:7 members

by selango » Sat May 29, 2010 8:53 am
From statement 1,the product of small and large number is positive.

The possible scenarios are like below,

S1={-1,-2,-3}, +ve

S2={1,2,3},+ve


If it contain both positive and negative integers,statement 1 will not be valid.

Ex:S={-1,2,3},the product is -ve

So Set should contain either all positive or all negative integers.

If the set of integers is as S2,it it sufficient. But for S1 since the number of integers are unknown,statement 1 is insufficient.


From statement 2,there are even number of integers in a set.So the possible scenarios,

S1={-1,2,3,4}=-ve

S2={1,2,3,4}=+ve


Since both scenarios are there statement 2 is insufficient.

Combining both,the conditions are,

1.The sets should contain either all positive integers or all negative integers.
2.There should be even number of intergers.

So the possible scenarios are,

S1={-1,-2,-3,-4}

S2={1,2,3,4}

since there are even number of integers,the product of all integers will be positive for both S1 and S2 and sufficient.

Hence answer is C

User avatar
GMAT Instructor
Posts: 1052
Joined: Fri May 21, 2010 1:30 am
Thanked: 335 times
Followed by:98 members

by Patrick_GMATFix » Tue Jun 08, 2010 5:39 am
[The reply below is copied from this thread]

To have a positive product, we must have an even number of negatives (0, 2, 4...) so that the negatives will cancel out in the multiplication.

REPHRASE: Are there an even number of negatives?

1) Max * Min is positive means Max and Min have the same sign. If they're both positive, then everything is positive and so is the product of all integers. However, if Max and Min are both negative, the product could be negative if we do NOT have an even number of negatives. Example {-3, -2, -1}. NOT SUFFICIENT.

2) By itself, this doesn't tell us whether there is an even number of negatives. Doesn't answer our rephrase.

Merge statements: (2) tells us that we have an even number of values. Since all the values have the same sign (1 says Max and Min have the same sign), either we have all positives or we have an even number of negatives. Either way, the product of all terms will be positive.

The answer is C

If the explanations above don't make sense to you, watch the step-by-step video solution. This is GMATPrep question 1106.

You can practice similar questions by using the Drill Engine to generate timed drills and setting topic='Number Properties' and difficulty='500-600 & 600-700'

Best of luck,
-Patrick
  • Ask me about tutoring.

Master | Next Rank: 500 Posts
Posts: 112
Joined: Wed Jan 20, 2010 5:46 am
Thanked: 1 times

by Thouraya » Fri Apr 22, 2011 8:02 am
SHOULD WE ASSUME THAT THE LIST OF NUMBERS IS IN ORDER? Otherwise, it doesn't make sense to choose C. Please some EXPERT help

User avatar
GMAT Instructor
Posts: 3225
Joined: Tue Jan 08, 2008 2:40 pm
Location: Toronto
Thanked: 1710 times
Followed by:614 members
GMAT Score:800

by Stuart@KaplanGMAT » Fri Apr 22, 2011 8:16 am
Thouraya wrote:SHOULD WE ASSUME THAT THE LIST OF NUMBERS IS IN ORDER? Otherwise, it doesn't make sense to choose C. Please some EXPERT help
Hi!

It doesn't matter if the list is in order. "Greatest" means the biggest number in the list and "smallest" means the smallest number in the list, regardless of how you arrange the terms. For example, in the sets:

{1, 2, 3, 4, 92}

and

{2, 4, 92, 3, 1}

the product of the greatest and smallest numbers is still 92*1 = 92.

I hope that clears up your confusion - if not, let me know exactly why you think ordering the list matters.
Image

Stuart Kovinsky | Kaplan GMAT Faculty | Toronto

Kaplan Exclusive: The Official Test Day Experience | Ready to Take a Free Practice Test? | Kaplan/Beat the GMAT Member Discount
BTG100 for $100 off a full course